Answered step by step
Verified Expert Solution
Link Copied!

Question

1 Approved Answer

Two rms, Firm 1 and Firm 2, compete by simultaneously choosing prices. Both rms sell an identical product for which each of 100 consumers has

image text in transcribed
image text in transcribed
Two rms, Firm 1 and Firm 2, compete by simultaneously choosing prices. Both rms sell an identical product for which each of 100 consumers has a maximum willingness to pay of 340. Each consumer will buy at most 1 unit, and will buy it from whichever rm charges the lowest price. If both rms set the same price, they share the market equally. Costs are given by Ci (qi) = lqi. Because of government regulation, rms can only choose prices which are integer numbers, and they cannot price above $40. Answer the following: a) (0.25 point) If Firm 1 chooses p1 = 27, Firm 2's best response is to set what price? [3 b) (0.25 point) If Firm 2 chooses the price determined in the previous question, Firm 1's best response is to choose what price? C] c) (1 point) If Firm 1 chooses p1 = 8. Firm 2's best response is a range of prices. What is the lowest price in this range? D

Step by Step Solution

There are 3 Steps involved in it

Step: 1

blur-text-image

Get Instant Access with AI-Powered Solutions

See step-by-step solutions with expert insights and AI powered tools for academic success

Step: 2

blur-text-image

Step: 3

blur-text-image

Ace Your Homework with AI

Get the answers you need in no time with our AI-driven, step-by-step assistance

Get Started

Recommended Textbook for

Accounting Tools for business decision making

Authors: kimmel, weygandt, kieso

4th Edition

978-0470117262, 9780470534786, 470117265, 470534788, 978-0470095461

Students also viewed these Economics questions

Question

What is your favorite restaurant in Kokshetau? Why? LO.1

Answered: 1 week ago

Question

How does an applicant apply?

Answered: 1 week ago